Question Stems Flashcards

1
Q

If the statements above are true, which one of the following must also be true?

A

Must Be True

How well did you know this?
1
Not at all
2
3
4
5
Perfectly
2
Q

If the information above is correct, which one of the following conclusions can be properly drawn on the basis of it?

A

Must Be True

How well did you know this?
1
Not at all
2
3
4
5
Perfectly
3
Q

The statements above, if true, most strongly support which one of the following?

A

Must Be True

How well did you know this?
1
Not at all
2
3
4
5
Perfectly
4
Q

Which one of the following can be properly inferred from the passage?

A

Must Be True

How well did you know this?
1
Not at all
2
3
4
5
Perfectly
5
Q

Which one of the following is most strongly supported by the information above, if they are true?

A

Must Be True

How well did you know this?
1
Not at all
2
3
4
5
Perfectly
6
Q

Which one of the following inferences is most supported by the information above?

A

Must Be True

How well did you know this?
1
Not at all
2
3
4
5
Perfectly
7
Q

If all of the statements above are true, which one of the following must be true?

A

Must Be True

How well did you know this?
1
Not at all
2
3
4
5
Perfectly
8
Q

Which one of the following conclusions is most strongly supported by the information above?

A

Must Be True

How well did you know this?
1
Not at all
2
3
4
5
Perfectly
9
Q

Which one of the following most accurately expresses the main conclusion of the argument?

A

Main Point

How well did you know this?
1
Not at all
2
3
4
5
Perfectly
10
Q

Which one of the following most accurately expresses the conclusion of the journalist’s argument?

A

Main Point

How well did you know this?
1
Not at all
2
3
4
5
Perfectly
11
Q

Which one of the following most accurately restates the main point of the passage?

A

Main Point

How well did you know this?
1
Not at all
2
3
4
5
Perfectly
12
Q

The main point of the argument is that:

A

Main Point

How well did you know this?
1
Not at all
2
3
4
5
Perfectly
13
Q

Which one of the following most accurately expresses the argument’s conclusion?

A

Main Point

How well did you know this?
1
Not at all
2
3
4
5
Perfectly
14
Q

The X’s reasoning provides grounds for accepting which one of the following statements?

A

Must Be True

How well did you know this?
1
Not at all
2
3
4
5
Perfectly
15
Q

The information above provides the LEAST support for which one of the following?

A

Must Be True - X

How well did you know this?
1
Not at all
2
3
4
5
Perfectly
16
Q

The main point of the argument is that:

A

Main Point

How well did you know this?
1
Not at all
2
3
4
5
Perfectly
17
Q

Which one of the following, if true, most seriously weakens the argument?

A

Weaken

How well did you know this?
1
Not at all
2
3
4
5
Perfectly
18
Q

Which one of the following, if true, most undermines the X’s argument?

A

Weaken

How well did you know this?
1
Not at all
2
3
4
5
Perfectly
19
Q

Which one of the following, if true, would most call into question the analyst’s explanation of the price increase?

A

Weaken

How well did you know this?
1
Not at all
2
3
4
5
Perfectly
20
Q

Which one of the following, if true, could be used by X to counter Y’s rejection of her explanation?

A

Weaken

How well did you know this?
1
Not at all
2
3
4
5
Perfectly
21
Q

Which one of the following, if true, is the strongest logical counter parent P can make to parent Q’s objection?

A

Weaken

How well did you know this?
1
Not at all
2
3
4
5
Perfectly
22
Q

Which one of the following, if true, most calls into question the claim above?

A

Weaken

How well did you know this?
1
Not at all
2
3
4
5
Perfectly
23
Q

Which one of the following, if true, most seriously weakens the X’s argument?

A

Weaken

How well did you know this?
1
Not at all
2
3
4
5
Perfectly
24
Q

Which one of the following, if true, casts the most doubt on the conclusion above?

A

Weaken

How well did you know this?
1
Not at all
2
3
4
5
Perfectly
25
Q

Each of the following, if true, weakens the X’s argument EXCEPT:

A

Weaken - X

How well did you know this?
1
Not at all
2
3
4
5
Perfectly
26
Q

The reasoning in the advertisement would be most strengthened if which one of the following were true?

A

Strengthen

How well did you know this?
1
Not at all
2
3
4
5
Perfectly
27
Q

Each of the following, if true, would strengthen the X’s argument EXCEPT:

A

Strengthen - X

How well did you know this?
1
Not at all
2
3
4
5
Perfectly
28
Q

Which one of the following, if true, LEAST strengthens the argument above?

A

Strengthen - X

How well did you know this?
1
Not at all
2
3
4
5
Perfectly
29
Q

Which one of the following discoveries, if it were made, would most support the above hypothesis about X and Y?

A

Strengthen

How well did you know this?
1
Not at all
2
3
4
5
Perfectly
30
Q

Which one of the following, if true, would most strengthen the X’s argument?

A

Strengthen

How well did you know this?
1
Not at all
2
3
4
5
Perfectly
31
Q

Each of the following, if true, strengthens the X’s argument EXCEPT:

A

Strengthen - X

How well did you know this?
1
Not at all
2
3
4
5
Perfectly
32
Q

Which one of the following, if true, most supports the argument?

A

Strengthen

How well did you know this?
1
Not at all
2
3
4
5
Perfectly
33
Q

The conclusion above follows logically if which one of the following is assumed?

A

Justify the Conclusion

How well did you know this?
1
Not at all
2
3
4
5
Perfectly
34
Q

Which one of the following, if assumed, would allow the conclusion to be properly drawn?

A

Justify the Conclusion

How well did you know this?
1
Not at all
2
3
4
5
Perfectly
35
Q

Which one of the following, if assumed, enables the argument’s conclusion to be properly inferred?

A

Justify the Conclusion

How well did you know this?
1
Not at all
2
3
4
5
Perfectly
36
Q

Which one of the following is an assumption that would serve to justify the conclusion above?

A

Justify the Conclusion

How well did you know this?
1
Not at all
2
3
4
5
Perfectly
37
Q

The X’s conclusion would be properly drawn if it were true that the:

A

Justify the Conclusion

How well did you know this?
1
Not at all
2
3
4
5
Perfectly
38
Q

The conclusion above is properly drawn if which one of the following is assumed?

A

Justify the Conclusion

How well did you know this?
1
Not at all
2
3
4
5
Perfectly
39
Q

The X’s conclusion follows logically if which one of the following is assumed?

A

Justify the Conclusion

How well did you know this?
1
Not at all
2
3
4
5
Perfectly
40
Q

Which one of the following is an assumption required by the argument above?

A

Assumption

How well did you know this?
1
Not at all
2
3
4
5
Perfectly
41
Q

Which one of the following is an assumption upon which the argument depends?

A

Assumption

How well did you know this?
1
Not at all
2
3
4
5
Perfectly
42
Q

The argument assumes which one of the following?

A

Assumption

How well did you know this?
1
Not at all
2
3
4
5
Perfectly
43
Q

The conclusion in the passage above relies on which one of the following assumptions?

A

Assumption

How well did you know this?
1
Not at all
2
3
4
5
Perfectly
44
Q

The position taken above presupposes which one of the following?

A

Assumption

45
Q

The conclusion cited does not follow unless:

A

Assumption

46
Q

Which one of the following is an assumption that the X’s argument requires in order for its conclusion to be properly drawn?

A

Assumption

47
Q

On which one of the following assumptions does the argument rely?

A

Assumption

48
Q

Which one of the following, if true, would most effectively resolve the apparent paradox above?

A

Resolve the Paradox

49
Q

Which one of the following, if true, most helps to resolve the apparent discrepancy in the passage above?

A

Resolve the Paradox

50
Q

Which one of the following, if true, most helps to explain the puzzling fact cited above?

A

Resolve the Paradox

51
Q

Which one of the following, if true, most helps to reconcile the discrepancy indicated above?

A

Resolve the Paradox

52
Q

Which one of the following, if true, most helps to resolve the apparent conflict described above?

A

Resolve the Paradox

53
Q

Which one of the following, if true, most helps to reconcile the X’s belief with the apparently contrary evidence described above?

A

Resolve the Paradox

54
Q

Which one of the following, if true, most helps to explain the finding of the study?

A

Resolve the Paradox

55
Q

Which one of the following, if true, contributes most to an explanation of the behavior of X described above?

A

Resolve the Paradox

56
Q

The method of the argument is to:

A

Method of Reasoning

57
Q

The argument proceeds by:

A

Method of Reasoning

58
Q

The argument derives its conclusion by:

A

Method of Reasoning

59
Q

Which one of the following describes the technique of reasoning used above?

A

Method of Reasoning

60
Q

Which one of the following is an argumentative strategy employed in the argument?

A

Method of Reasoning

61
Q

The argument employs which one of the following reasoning techniques?

A

Method of Reasoning

62
Q

A responds to B’s argument by:

A

Method of Reasoning

63
Q

The argument proceeds by:

A

Method of Reasoning

64
Q

The claim that people have positive or negative responses to many nonsense words plays which one of the following roles in the argument?

A

Method of Reasoning

65
Q

Which one of the following most accurately describes the role played in the psychologist’s argument by the claim that the obligation to express gratitude cannot be fulfilled anonymously?

A

Method of Reasoning

66
Q

The claim that humans are still biologically adapted to a diet of wild foods plays which one of the following roles in the nutritionist’s argument?

A

Method of Reasoning

67
Q

The phrase “XYZ” serves which one of the following functions in the argument?

A

Method of Reasoning

68
Q

Which one of the following most accurately describes a flaw in the argument’s reasoning?

A

Flaw in the Reasoning

69
Q

The reasoning in the argument is most vulnerable to criticism on the ground that the argument:

A

Flaw in the Reasoning

70
Q

The reasoning above is flawed because it fails to recognize that:

A

Flaw in the Reasoning

71
Q

A questionable aspect of the reasoning above is that it:

A

Flaw in the Reasoning

72
Q

The reasoning in the argument is fallacious because the argument:

A

Flaw in the Reasoning

73
Q

Which one of the following is a questionable argumentative strategy employed in the editorial’s argument?

A

Flaw in the Reasoning

74
Q

The argument is most vulnerable to criticism on which one of the following grounds?

A

Flaw in the Reasoning

75
Q

The reasoning in X’s argument is flawed because that argument:

A

Flaw in the Reasoning

76
Q

The reasoning in the argument is flawed in that the argument:

A

Flaw in the Reasoning

77
Q

Which one of the following is most closely parallel in its reasoning to the reasoning in the argument above?

A

Parallel Reasoning

78
Q

Which one of the following exhibits a pattern of reasoning most similar to that exhibited by the argument above?

A

Parallel Reasoning

79
Q

Which one of the following arguments is most similar in its logical features to the argument above?

A

Parallel Reasoning

80
Q

Which one of the following arguments is most similar in its pattern of reasoning to the argument above?

A

Parallel Reasoning

81
Q

The structure of the reasoning in the argument above is most parallel to that in which one of the following?

A

Parallel Reasoning

82
Q

Which one of the following exhibits a flawed pattern of reasoning most similar to that in the argument above?

A

Parallel Reasoning

83
Q

The pattern of reasoning displayed in the argument above is most closely paralleled by that in which one of the following arguments?

A

Parallel Reasoning

84
Q

Which one of the following exhibits both of the logical flaws exhibited but he argument above?

A

Parallel Reasoning

85
Q

The answer to which one of the following questions would contribute most to an evaluation of the argument?

A

Evaluate the Argument

86
Q

Clarification of which one of the following issues would be out important to an evaluation of the skeptic’s position?

A

Evaluate the Argument

87
Q

Which one of the following would be most important to know in evaluating the hypothesis in the passage?

A

Evaluate the Argument

88
Q

Which one of the following would it be most relevant to investigate in evaluating the conclusion of X’s argument?

A

Evaluate the Argument

89
Q

Which one of the following would it be most helpful to know in order to judge whether what the scientist subsequently learned calls into question the hypothesis?

A

Evaluate the Argument

90
Q

The answer to which one of the following questions would be most useful in evaluating the truth of the conclusion drawn in the advertisement?

A

Evaluate the Argument

91
Q

If the statements above are true, which one of the following CANNOT be true?

A

Cannot Be True

92
Q

The argument can most reasonably be interpreted as an objection to which one of the following claims?

A

Cannot Be True

93
Q

The statements above, if true, most seriously undermine which one of the following assertions?

A

Cannot Be True

94
Q

The information above, if accurate, can best be used as evidence against which one of the following hypotheses?

A

Cannot Be True

95
Q

If all of the claims made above are true, then each of the following could be true EXCEPT:

A

Cannot Be True

96
Q

If the statements above are true, then which one of the following must be false?

A

Cannot Be True

97
Q

Which one of the following most accurately expresses the point at issue between X and Y?

A

Point at Issue

98
Q

Which one of the following most accurately represents what is at issue between X and Y?

A

Point at Issue

99
Q

The dialogue above lends the most support to the claim that X and Y disagree with each other about which one of the following statements?

A

Point at Issue

100
Q

On the basis of their statement, X and Y are committed to disagreeing over whether:

A

Point at Issue

101
Q

X and Y’s statements provide the most support for holding that they disagree about the truth of which one of the following?

A

Point at Issue

102
Q

X and Y disagree over whether:

A

Point at Issue

103
Q

Each of the following principles is logically consistent with the columnist’s conclusion EXCEPT:

A

Cannot - PR

104
Q

Which one of the following most accurately expresses the principle underlying the argumentation above?

A

Justify - PR

105
Q

The information above most closely conforms to which one of the following principles?

A

Strengthen - PR

106
Q

Which one of the following principles most helps to justify the reasoning above?

A

Strengthen - PR

107
Q

The principles above, if established, would justify which one of the following judgments?

A

Must - PR

108
Q

Which one of the following judgments best illustrates the principle illustrated by the argument above?

A

Must - PR

109
Q

Which one of the following judgments most closely conforms to the principle above?

A

Must - PR